Download as pdf or txt
Download as pdf or txt
You are on page 1of 29

UPSCMATERIAL.

ONLINE

Contact us : info@onlyias.com

OnlyIAS Nothing Else Visit : dpp.onlyias.in


Contact : +91-7007 931 912

Q.1) Consider the following statements: a) 1 only


1. The Warsaw Pact was created in reaction to the b) 2 only
integration of West Germany into NATO. c) Both 1 and 2
2. India became a member of the Warsaw Pact with d) Neither 1 nor 2
the Indo-Soviet Treaty of Friendship in 1971.
Which of the above statement is/are correct? Q.5) With reference to the important Bandung
a) 1 only Conference of 1955, consider the following
b) 2 only statements.
c) Both 1 and 2 1. It was organized to promote Afro Asian economic
d) Neither 1 nor 2 and cultural cooperation.
2. The first Africa-India Summit was organized on
Q.2) With reference to the New International the sidelines of the conference.
Economic Order (NIEO), often mentioned in the post- Which of the above statement is/are correct?
colonial narratives, consider the following statements. a) 1 only
1. NIEO was a demand raised by WTO developed b) 2 only
country members for further liberalization of c) Both 1 and 2
global trade regime. d) None
2. It called for strengthening market forces and
rationalization of the bureaucracy. Q.6) The term Multi-Polarity often heard in the
3. It envisaged that developing countries must be context of international relations refers to
entitled to regulate and control the activities of 1. It is related to Success of democracy in modern
multinational corporations operating within their world.
territory. 2. It envisage to Balance of power in the world.
Select the correct option using code below. Which of the above Statement is/are correct?
a) 1 and 2 only a) 1 only
b) 2 and 3 only b) 2 only
c) 1 and 3 only c) Both 1 and 2
d) 1,2 and 3 d) Neither 1 nor 2

Q.3) Consider the following about Association of Q.7) Choose the correct statements about reasons for
South East Asian Nations (ASEAN). acceleration of globalization:
1. The East Asia Summit involves all the ASEAN 1. The invention of the telegraph, telephone and
members. microchip has revolutionized communication.
2. It has an observer status at the United Nations 2. Printing came into being laying the basis for the
General Assembly. creation of nationalism.
3. The ASEAN Way refers to a multi-modal trade 3. Technology affects not only the way we think of
corridor developed by ASEAN along the old Silk our personal but also our collective lives.
Route. Which of the following statements is/are correct?
Select the correct answer using the codes below. a) 1 only
a) 1 and 2 only b) 1 and 2 only
b) 1 only c) 2 and 3 only
c) 2 and 3 only d) 1 ,2 and 3
d) 3 only
Q 8) World Social Forum is a global platform opposed
Q.4) With reference to Amnesty International, to neoliberal globalization. Which of the following
consider the following statements: statements about WSF is/are incorrect?
1. It conceptualized and introduced the Universal 1. It is a wide coalition composed of human rights
Declaration of Human Rights in the United activists, environmentalists, labour, youth and
Nations General Assembly. women activists.
2. It is the oldest working international organization 2. The first WSF meeting was organized in Geneva,
to uphold human rights in the world. Switzerland.
Which of the above Statement is/are correct?

@upscmaterialonline
NCERT CLASS XII CONTEMPORARY WORLD POLITICS ENGLISH Page 1
UPSCMATERIAL.ONLINE

Contact us : info@onlyias.com

OnlyIAS Nothing Else Visit : dpp.onlyias.in


Contact : +91-7007 931 912

3. Fourth and seventh WSF meeting were held in Q.11) Consider the following statements about origin
Delhi. of European union, Which of the following is/are not
Select the correct?
correct option 1. European integration after 1947 was aided by the
using code Cold War.
given below: 2. The Organisation for European Economic
a) 1 only Cooperation was established in 1948 under the
b) 1 and 2 only Marshall Plan to aid east European states.
c) 2 and 3 only 3. The Council of Europe was established in 1948 for
d) 1 ,2 and 3 political cooperation.
Select the correct option using code given below:
Q.9) The United Nations Conference on Trade and a) 1 only
Development (UNCTAD) brought out a report Towards b) 1 and 2 only
a New Trade Policy for Development. c) 2 and 3 only
1. Give the LDCs control over their natural resources d) 1 ,2 and 3
exploited by the developed Western countries.
2. Obtain access to Western markets so that the Q.12) The Union of Soviet Socialist Republics (USSR)
LDCs could sell their products for making trade came into being after the socialist revolution in Russia
more beneficial for poorer countries. in 1917. Which of the following statements is/are
3. It increases the cost of technology from the correct in this regard?
Western countries. 1. The revolution was inspired by the ideals of
4. Provide the LDCs with a greater role in socialism as opposed to capitalism and the need
international economic institutions. for an egalitarian society.
Which of the 2. The makers of the Soviet system gave primacy to
following the state and the institution of the party.
statements 3. This was perhaps the biggest attempt in human
is/are correct? history to abolish the institution of private
a) 1 and 4 property and consciously design a society based
only on principles of equality.
b) 1, 2 and 4 only Select the correct option using code given below:
c) 2 and 3 only a) 1 and 2 only
d) 1 ,2 and 3 b) 3 only
c) 2 and 3 only
Q.10) Consider the following statements with d) 1,2 and 3
reference to the disintegration of Soviet Union:
1. A coup took place in 1991 encouraged by Q.13) Consider the following statement about ASEAN
Communist Party hardliners. and choose the incorrect one/s.
2. Boris Yeltsin emerged as a national hero in 1. ASEAN Community comprises of three pillars
opposing this coup. including ASEAN Military Community and ASEAN
3. In December 1991, under the leadership of Yeltsin Socio-Cultural Community.
Russia, Ukraine and Belarus declared that the 2. The ASEAN Regional Forum was established in
Soviet Union was disbanded. 1994 to carry out coordination of security and
Which of the foreign policy.
following Select the correct option using code given belo
statements a) 1 only
is/are correct? b) 2 only
a) 1 only c) Both 1 and 2
b) 1 and 2 only d) Neither 1 nor 2
c) 2 and 3 only
d) 1 ,2 and 3 Q.14) Consider the following statements about
relation between India and it’s neighbours.

@upscmaterialonline
NCERT CLASS XII CONTEMPORARY WORLD POLITICS ENGLISH Page 2
UPSCMATERIAL.ONLINE

Contact us : info@onlyias.com

OnlyIAS Nothing Else Visit : dpp.onlyias.in


Contact : +91-7007 931 912

1. India and Nepal had conflicts related to migration 3. There was a consensus between developed and
of ethnic Napalese into India. developing countries on combining economic
2. In 1960 with the help of the World Bank India and growth with ecological responsibility.
Pakistan signed the Indus Waters Treaty. Which of the
Which of these statements are not correct? following
a) 1 only statements
b) 2 only is/are correct?
c) Both 1 and 2 a) 1 and 3
d) Neither 1 nor 2 only
b) 1 and 2 only
Q.15) The South Asian Association for Regional c) 2 and 3 only
Cooperation (SAARC) is a major regional initiative by d) 1 ,2 and 3
the South Asian states. Which among the following
statements is/are correct in this regard? Q.18) What is often called economic globalization
1. SAARC was comparatively successful in most of usually involves greater economic flows among
the arenas inspite of the political differences different countries of the world. Choose the correct
between the member countries. statements from the following about the features of
2. External influence of non-regional players is a economic globalisation.
major bone of contention which prevents 1. The restrictions imposed by different countries on
attainment of full potential of SAARC. allowing the imports of other countries have been
Select the correct option using code given below: reduced.
a) 1 only 2. The restrictions on movement of capital across
b) 2 only the countries have been increased.
c) Both 1 and 2 3. Developed countries carefully guarded their
d) Neither 1 nor 2 borders with visa policies to ensure that citizens
of other countries cannot take away the jobs of
Q.16) Contemporary World Politics have expanded its their own citizens.
scope to include environmental concerns as the major Select the correct option using code given below:
element in global politics today. Which of the a) 1 and 3 only
following statement/s is are correct regarding this? b) 2 and 3 only
1. Throughout the world cultivable area is barely c) 1 only
expanding and a substantial portion of existing d) All the above
agricultural land is losing fertility.
2. There is a steady decline in the total amount of Q.19) International Monetary Fund (IMF) is an
ozone in the Earth’s stratosphere. international organization that oversees those
3. Coastal waters are relatively less polluted than financial institutions and regulations that act at the
the open sea. international level. Consider the following statements
Select the correct option using code given below: and choose the correct one/s.
a) 1 and 2 only 1. The IMF has 190 member countries but they do
b) 2 and 3 only not enjoy an equal say.
c) 3 only 2. The IMF promotes international monetary co-
d) All of the above operation and exchange rate stability.
3. IMF is a United Nations specialized agency.
Q.17) Consider the following statements regarding Select the correct option using code given below:
Earth summit: a) 1 only
1. The Summit was attended by 170 states b) 1 and 2 only
thousands of NGOs and many multinational c) All of the above
corporations. d) 1 and 3 only
2. The 1987 Brundtland Report Our Common Future
acted as a precursor to Earth Summit. Q.20) Which among the following gives more
weightage to India’s proposal for permanent
membership in the Security Council.

@upscmaterialonline
NCERT CLASS XII CONTEMPORARY WORLD POLITICS ENGLISH Page 3
UPSCMATERIAL.ONLINE

Contact us : info@onlyias.com

OnlyIAS Nothing Else Visit : dpp.onlyias.in


Contact : +91-7007 931 912

a) India’s growing economic power and stable statements


political system. is/are correct?
b) Nuclear capability. a) 1 only
c) It has been a member of UN since its inception. b) 1 and 2 only
d) It is located in Asia. c) 2 and 3 only
d) 1 ,2 and 3
Q.21) The Millennium Development Goals (MDGs) are
eight international development goals that were Q.24) Consider the following statements about
established following the Millennium Summit of UN in Maastricht Treaty, based on which the European
2000. Which among the following are Millennium Union was formed. Choose the correct statements.
Development Goals? 1. The treaty provides for single visa for all the
1. To achieve Universal Primary Education European Union countries.
2. To improve Maternal Health 2. Margaret Thatcher kept the UK out of the
3. Provide safe Drinking Water European market.
4. To improve Sanitation 3. Denmark and Sweden have resisted the
5. To develop Global Partnership for Development Maastricht treaty and the adoption of the euro.
6. To promote Gender Equality and Empower Which of the
Women. following
statements
Select the correct option using code given below: is/are correct?
a) 1, 2, 5 and 6 only a) 1 only
b) 1, 2 and 6 only b) 1 and 2
c) 5 and 6 only only
d) 2, 3 and 4 only c) 2 and 3
only
Q.22) Which of the following were used by US to d) 1 ,2 and 3
establish its hegemony over the world?
1. Military domination Q.25) Consider the following statements about the
2. Economic power formation of ASEAN:
3. Political clout 1. ASEAN was established in1967 by ten countries of
4. Cultural Superiority the region through the Singapore declaration.
5. Establishment of dictatorship 2. The primary objective of the association is to
Which of the following statements is/are correct? accelerate economic growth and through that
a) 1,2,3 only ‘social progress and cultural development’.
b) 1,2,3,4 only 3. A secondary objective was to promote regional
c) 1,3,4,5 only peace and stability based on the rule of law and
d) 1,2,3,4 and 5 the principles of the United Nations Charter.
Which of the
Q.23) Hegemony is also about the capacity to following
‘manufacture consent’. Consider the following statements
statements in this regard and choose the correct is/are correct?
one/s. a) 1 only
1. Hegemony implies class ascendancy in the social, b) 1 and 2 only
political and particularly ideological spheres. c) 2 and 3 only
2. Hegemony arises when the dominant class or d) 1 ,2 and 3
country can win the consent of the dominated
classes. Q.26) Consider the following statements with respect
3. It is achieved by coercing the dominated classes to United nations security council (UNSC):
to view the world in a manner favourable to the 1. The council consists of 15 members including 5
ascendancy of the dominant class. permanent and 5 non permanent members.
Which of the 2. UNSC was established by the UN Charter in 1945.
following

@upscmaterialonline
NCERT CLASS XII CONTEMPORARY WORLD POLITICS ENGLISH Page 4
UPSCMATERIAL.ONLINE

Contact us : info@onlyias.com

OnlyIAS Nothing Else Visit : dpp.onlyias.in


Contact : +91-7007 931 912

Which of the following statements is/are correct Q.30) Consider the following statements with respect
to United nations ( UN) :
a) 1 only
b) 2 only 1. The name United Nations was coined by United
c) Both 1 and 2 States President Franklin D. Roosevelt.
d) Neither 1 nor 2 2. It has Six principal organs and all were established
in the same founding year.
Q.27) With reference to the International
Organisations , which of the following statement is Which of the following statements is/are correct?
correct? a) 1 only
1. An international organisation is a super state with b) 2 only
authority over its members. c) Both 1 and 2
2. It is created by and responds to states. d) Neither 1 nor 2
3. It comes into being states agrees to its creation.
Select the correct option using code below. Q.31) Consider the following statements with
reference to International court of justice (ICJ) , which
a) 1 and 2 only of the following statement is incorrect?
b) 2 and 3 only 1. It was established by the untied nations charter in
c) 1 and 3 only 1946.
d) 1,2 and 3 2. It is principal organ of UN and placed in New York.
3. It consists of 15 judges elected for the term of
Q.28) Consider the following statements with respect three years by UN.
to International monetary Fund( IMF):
Select the correct option using code below.
1. It is an international organisation that
oversees those financial institutions and a) 1 and 2 only
regulation that act at the international level b) 2 and 3 only
and national level. c) 1 and 3 only
2. It has 189 members and they enjoy an equal say. d) 1,2 and 3
Which of the following statements is/are incorrect? Q.32) Consider the following statements with respect
to International court of justice (ICJ):
a) 1 only
b) 2 only 1. ICJ is assisted by Registry and its official language
c) Both 1 and 2 is English only.
d) Neither 1 nor 2 2. Similar to the other organs of international
organizations it is composed of representatives of
Q.29) With reference to the evolution of United governments.
Nations( UN) , which of the following statement is
correct? Which of the following statements is/are correct?
1. The UN was founded as a successor of the League a) 1 only
of Nations. b) 2 only
2. It was established immediately after the first c) Both 1 and 2
world war. d) Neither 1 nor 2
3. It has an intention to bring countries together to
improve social and economic conditions of the Q.33) Consider the following statements with
world. reference to the Trusteeship council (TC) ,which of
the following statement is correct?
Select the correct option using code below.
1. 1.Trust territory is a self-governing territory under
a) 1 and 2 only an administrative authority by the Trusteeship
b) 2 and 3 only Council.
c) 1 and 3 only 2. 2.United Nations trust territories were the
d) 1,2 and 3 successors of the remaining League of Nations.

@upscmaterialonline
NCERT CLASS XII CONTEMPORARY WORLD POLITICS ENGLISH Page 5
UPSCMATERIAL.ONLINE

Contact us : info@onlyias.com

OnlyIAS Nothing Else Visit : dpp.onlyias.in


Contact : +91-7007 931 912

Select the correct option using code below. d) Neither 1 nor 2


a) 1 only
b) 2 only Q.37) Which of the following steps taken to make UN
c) Both 1 and 2 more relevant in the changing world?
d) Neither 1 nor 2
1. Agreements to achieve the Millennium
Q.34) With reference to the United Nations General Development Goals.
Assembly , which of the following statement is 2. Acceptance of the responsibility of the
correct? international community in case of failures of
national governments to protect their own
1. All the members of the UN are represented in the citizens from atrocities.
UNGA.
2. 2.It meets annually in its headquarters in Geneva. Select the correct option using code below.
3. It is also known as parliament of the world
a) 1 only
Select the correct option using code below. b) 2 only
c) Both 1 and 2
a) 1 and 2 only d) Neither 1 nor 2
b) 2 and 3 only
c) 1 and 3 only
d) 1,2 and 3
Q.38) With reference to the Amnesty international,
consider the following statements:
Q.35) Consider the following statements with respect
to Economic and Social council (ECOSOC): 1. It works Under supervision of United Nations.
2. It promotes respect for all the human rights in the
1. It is a principle body for coordination and universal declaration of human rights.
recommendations on economic, social and 3. It prepares and publishes report on human rights.
environmental issues.
2. Food and agricultural organisation and Select the correct option using code below.
International labour organisation falls under a) 1 and 2 only
purview of ECOSOC. b) 2 and 3 only
Which of the following statements is/are correct? c) 1 and 3 only
d) 1,2 and 3
a) 1 only
b) 2 only
c) Both 1 and 2 Q.39) Consider the following statements with
d) Neither 1 nor 2 reference to International Atomic Energy Agency:
1. The primary mandate of the organisation was and
Q.36) With reference to the UN General Assembly continues to be promoting safe, secure and
peaceful nuclear technologies.
resolution, which of the following statement is
correct? 2. IAEA teams regularly inspect nuclear facilities
all over the world to ensure that civilian re ac
1. The Security Council no longer represents tors are not being used for military purposes.
contemporary political realities.
2. Its decisions reflect Global values and interests Select the correct option using code below.
and are dominated by all countries. a) 1 only
Select the correct option using code below. b) 2 only
c) Both 1 and 2
a) 1 only d) Neither 1 nor 2
b) 2 only
c) Both 1 and 2

@upscmaterialonline
NCERT CLASS XII CONTEMPORARY WORLD POLITICS ENGLISH Page 6
UPSCMATERIAL.ONLINE

Contact us : info@onlyias.com

OnlyIAS Nothing Else Visit : dpp.onlyias.in


Contact : +91-7007 931 912

Q.40) Consider the following statements with respect 2. There is no provision of impeachment for a
to World Trade organization( WTO): Governor as in the case of the President.
3. Similar to the President, the Governor enjoys
1. This organization was set up in 1995 as the diplomatic and military powers.
successor to the General Agreement on Trade Select the correct answer using the codes below.
and Tariffs created after the Second World War a) 1 and 3 only
2. Major economic powers US, EU and Japan have b) 2 only
managed to use the WTO to frame rules in c) 1, 2 and 3
concern with developing countries. d) 2 and 3 only
Which of the following statements is/are correct?
Q.44) Which of the following is/are the key features
a) 1 only of Indian Federalism?
b) 2 only 1. The existence and authority of each tier of
c) Both 1 and 2 government is constitutionally guaranteed.
d) Neither 1 nor 2 2. Division of powers between State and Centre
Q.41) The term ‘Lack of Internal democracy’ is cannot be arbitrarily manipulated by the Central
sometimes heard in the context of Indian political government alone.
parties. The term implies that Select the correct answer using the codes below.
1. Power is concentrated at the top and a) 1 only
participatory decision-making is largely absent. b) 2 only
2. There are no provincial or local branches of the c) Both 1 and 2
party and workers are drawn from the Central d) None of the above
pool only.
3. There is no constitutional document for the party. Q.45) Which of the following statements is correct
Which of the above is/are correct? with reference to the constitution of India?
a) 1 only 1. It is a set of written rules that are accepted by all
b) 2 and 3 only people living together in a country.
c) 1 and 3 only 2. It specifies how the government will be
d) 1, 2 and 3 constituted and the distribution of decision-
making power therein.
Q.42) Consider the following statements. 3. It expresses the aspirations of the people about
1. A whip in parliamentary parlance is a written creating a good society.
order that party members be present for an Select the correct answer using the codes below.
important vote, or that they vote only in a a) 1 only
particular way. b) 2 and 3 only
2. In India, only national/state parties can issue a c) 1 and 3 only
whip to their members. d) 1, 2 and 3
3. The anti-defection law allows the
Speaker/Chairperson to disqualify such a member Q.46) The powers and the independence of the Indian
who goes against the voting directions of the judiciary allow it to act as the guardian of Indian
whip. democracy. What are these powers?
Select the correct answer using the codes below. 1. Anyone can approach the higher judiciary if public
a) 1 and 2 only interest is hurt by the actions of government.
b) 2 and 3 only 2. Higher Judiciary can issue directives to check
c) 1 and 3 only malpractices on the part of public officials.
d) 1, 2 and 3 Select the correct option using code given below:
a) 1 only
Q.43) Consider the following statements: b) 2 only
1. The same person cannot be the Governor of two c) Both 1 and 2
or more states. d) Neither 1 nor 2

@upscmaterialonline
NCERT CLASS XII CONTEMPORARY WORLD POLITICS ENGLISH Page 7
UPSCMATERIAL.ONLINE

Contact us : info@onlyias.com

OnlyIAS Nothing Else Visit : dpp.onlyias.in


Contact : +91-7007 931 912

Q.47) Adaptation fund, for financing projects and b) 1 only


programmes that help vulnerable communities in c) 1 and 3 only
developing countries adapt to climate change is d) All of the above
established under which among the following?
a) Paris Agreement Q.52) Consider the following statements:
b) Montreal Protocol 1. Japan has very few natural resources and imports
c) Kyoto Protocol most of its raw materials.
d) Kigali Agreement 2. It is the only Asian member of the G-7.
3. It is the second largest contributor to the regular
Q.48) Consider the following budget of the UN.
statements about Intergovernmental 4. Japan’s military expenditure is the fourth largest
Panel on Climate Change (IPCC) in the world.
1. The Intergovernmental Panel on Climate Change Which of the following statements is correct?
is the United Nations body for assessing the a) 1 and 2 only
science related to climate change. b) 2 and 3 only
2. 2.The IPCC does not conduct its own research. c) 1, 2 and 3 only
3. It was created by UNDP in 1988 d) All of the above
Which of the above statements is/are correct?
a) 1 and 2 only Q.53) The invasion and annexation of Kuwait by Iraq
b) 2 and 3 only lead to the First Gulf War. Consider the following
c) 1 and 3 only statement:
d) 1, 2 and 3 1. There were a series of diplomatic attempts to
convince Iraq to quit its aggression.
Q.49) Which among the following serves as the 2. The resolution for this war got defeated in UN but
trustee of the Global Environment Facility? they silently supported this war.
a) UNEP 3. America led a coalition of forces from 34
b) World Bank countries and Iraq got defeated quickly.
c) UNFCCC Which of the following statements is correct?
d) IUCN a) 1 only
b) 2 and 3 only
Q.50) Which among the following is NOT a member of c) 1 and 3 only
ADMM Plus countries? d) 2 only
a) USA
b) Russia Q.54) Consider the following statements in the
c) India context of NAM and choose incorrect statements:
d) Germany 1. The New International Economic Order was a set
of proposals put forward during the 1970s by
Q.51) Consider the following statements about the some developing countries through the United
condition of China before they started the economic Nations Conference on Trade and Development.
reform process. Which of the following statement is 2. It was meant to be a revision of the international
correct? economic system in favour of Third World
1. From the inception of the People’s Republic of countries replacing the Bretton Woods system.
China in 1949 they were following the economy 3. NIEO initiative faded in late 1980s in wake of stiff
based on Soviet model. opposition by developed countries and
2. Agricultural production generated surplus for nonaligned countries struggled to maintain their
China to start the industries in a better way unity.
compared to India. Select the correct option using the code given below:
3. Before reforms China’s industrial production was a) 1 only
not growing fast enough to play a good role in b) 1 and 2 only
international trade. c) 3 only
Select the correct option using code below: d) None of the above
a) 1 and 2 only

@upscmaterialonline
NCERT CLASS XII CONTEMPORARY WORLD POLITICS ENGLISH Page 8
UPSCMATERIAL.ONLINE

Contact us : info@onlyias.com

OnlyIAS Nothing Else Visit : dpp.onlyias.in


Contact : +91-7007 931 912

Q.55) Consider the following statements regarding 3. Non-recognition of cultural affairs of people
the Arms Control Treaties which have been signed in residing in these republics.
International arena during the post 2nd World War era 4. Soviet invasion of Afghanistan.
and which of the following statement is incorrect? Select the correct option using code below:
1. LTBT (Limited Test Ban Treaty) banned nuclear a) 1, 2 and 3 only
weapon tests exclusively in atmosphere. b) 2, 3 and 4 only
2. NPT was signed in 1968 entered into force in 1970 c) 1, 3 and 4 only
and extended indefinitely in 1995. d) All of the above
Select the correct option using code below:
a) 1 only Q.59) There are some important events which
b) 2 only happened in course of disintegration of Soviet Union.
c) Both 1 and 2 Consider the following statements :
d) Neither 1 nor 2 1. Latvia becomes first of 15 Soviet republics to
declare its independence.
Q.56) Inspite of cold war USA and USSR was vary of 2. Permission for Multi-party politics was given in
real war and many arms control treaties were signed 1990.
between USA and USSR/Russia. Consider the 3. Russia, Belarus and Armenia established
following statements: Commonwealth of Independent States in 1991.
1. Treaty on limitation of Anti-ballistic missile system Select the correct option using code below:
was signed in 1972 during Strategic Arms a) 1 and 2 only
Limitation Talk’s second round (SALT-II) b) 2 and 3 only
2. Strategic arms reduction treaty-I was signed c) 2 only
between Russian President Boris Yeltsin and US d) 3 only
President George Bush in 1993.
Which of the following statements is correct? Q.60) New theories and doctrines were proposed
a) 1 only during cold war era to control the influence of
b) 2 only opposing bloc. Consider the following statements
c) Both 1 and 2 regarding these:
d) Neither 1 nor 2 1. The Truman Doctrine of containment was a
United States policy to stop Soviet expansion
Q.57) There had been many important events during during the Cold War.
cold war era. Arrange these events in chronological 2. The Marshall Plan was the American initiative to
order: aid Europe.
1. Korean war resulting in division of Korea Which of the following statement is correct?
2. Signing of the Baghdad pact a) 1 only
3. American intervention in Vietnam b) 2 only
4. Cuban missile crisis c) Both 1 and 2
5. Formation of SEATO d) Neither 1 nor 2
Select the correct option using code below:
a) 1-5-3-2-4 Q.61) In Eastern Europe, there have been splits in the
b) 1-5-2-3-4 countries and emergence of new countries as
c) 5-1-3-4-2 communism was weakened due to various causes.
d) 1-5-4-2-3 Consider the following statements:
1. Yugoslavia went a peaceful split resulting in
Q.58) Soviet system came into being after socialist emergence of states like Slovenia, Bosnia and
revolution in 1917 but it disintegrated in 1990s , Herzegovina.
which were the following reasons for its 2. Czechoslovakia split resulted into a major conflict
disintegration? and Czechs and Slovaks formed two independent
1. One-party system. countries.
2. Russian dominance over other 14 republics of Which of the following statements is incorrect?
USSR. a) 1 only
b) 2 only

@upscmaterialonline
NCERT CLASS XII CONTEMPORARY WORLD POLITICS ENGLISH Page 9
UPSCMATERIAL.ONLINE

Contact us : info@onlyias.com

OnlyIAS Nothing Else Visit : dpp.onlyias.in


Contact : +91-7007 931 912

c) Both 1 and 2 d) All of the above


d) Neither 1 nor 2
Q.65) United Nations Security Council (UNSC) is one of
Q.62) Economic policies of post communist regime the six principal organs of the UN and are charged
also underwent a change after the disintegration of with the maintenance of International Peace and
Soviet Union. Consider the following statement: Security. Consider the following :
1. Initiatives were taken to attract foreign 1. Security Council have five permanents members
investment along with currency convertibility and and ten other members elected by the General
deregulation policies. Assembly for three year terms.
2. Trade alliances with Western countries were 2. The decision of Security Council is binding on all
made besides the alliances among the Soviet Bloc UN members except UNSC permanent members.
countries. Which of the following statement is incorrect?
3. Western capitalist states controlled the a) 1 only
development of these countries. b) 2 only
Which of the following statements is correct? c) Both 1 and 2
a) 1 and 2 only d) Neither 1 nor 2
b) 2 and 3 only
c) 1 and 3 only Q.66) Consider the following statements regarding
d) None of the above Organization for the Prohibition of Chemical Weapons
(OPCW) :
Q.63) After the disintegration of USSR newly 1. OPCW is an inter governmental organization
independent countries started reviving after about located in Brussels.
ten years of decline. The reasons cited for their revival 2. The organization promotes and verifies the
are: adherence to the chemical weapons only.
1. Revival was brought by export of oil and natural 3. OPCW is not an agency of the United Nations.
gas. Which of the following statement is incorrect?
2. Rent gained from oil pipelines crossing many a) 1 only
countries. b) 2 and 3 only
3. Priority given to establishment of stable c) 1 and 2 only
democratic institutions. d) 3 only
4. Independence of judiciary, prevented any dissent
among these countries. Q.67) World Bank was established under Bretton
Which of the following statements is correct? woods agreement after the Second World War.
a) 1 and 2 only Choose the incorrect statements about World Bank:
b) 2, 3 and 4 only 1. International Development Association is the
c) 1, 2 and 4 only investment arm of World Bank which provides
d) All of the above loans to private sector.
2. International Finance Commission is the soft loan
Q.64) Post USSR disintegration, geopolitical scenario window of World Bank.
has undergone change between India and USSR. 3. Multilateral Investment Guarantee Agency (MIGA)
Consider the following statements regarding this: is the insurance arm of World Bank.
1. India has strong friendly relations with Russia but Select the correct option using code below:
it is not quiet friendly with other Soviet bloc a) 1 and 2 only
countries after their independence. b) 2 and 3 only
2. Soviet Union gave assistance to Indian public c) 1 and 3 only
sector companies. d) All of the above
3. India has remained a major arms and oil importer
from Russia. Q.68) World Trade Organisation (WTO) is an
Which of the following statement is correct? international organization that sets rules for global
a) 1 and 2 only trade. Which of the following statements about WTO
b) 1 and 3 only is/are correct?
c) 2 and 3 only

@upscmaterialonline
NCERT CLASS XII CONTEMPORARY WORLD POLITICS ENGLISH Page 10
UPSCMATERIAL.ONLINE

Contact us : info@onlyias.com

OnlyIAS Nothing Else Visit : dpp.onlyias.in


Contact : +91-7007 931 912

1. It was set up in 1995 as the successor to General b) 2 only


Agreement on Tariffs and Trade. c) Both 1 and 2
2. WTO has 160 members with Yemen being the d) Neither 1 nor 2
latest to join on June 26, 2014.
3. The decision making power is WTO mostly lies Q.72) With reference to the India china relationship,
with the top 10 countries having 55% of the votes. Consider the following statements:
Select the correct option using code below: 1. China had considerable influence and control on
a) 1 and 2 only the periphery of its borders and tributary system.
b) 1 and 3 only 2. After the change in China’s political leadership
c) 2 and 3 only from in mid 1970s China’s policy became less
d) All of the above pragmatic and more ideological.
Which of the following statements is/are correct?
Q.69) Consider the following statements about a) 1 only
SAARC: b) 2 only
1. The SAARC is a major regional initiative by South c) Both 1 and 2
Asian states to evolve cooperation through d) Neither 1 nor 2
multilateral means.
2. SAARC members signed the south Asian free trade Q.73) Consider the following statements Regarding
agreement which promised the formation of a Lebanon crisis:
trade zone for whole zone of South Asia. 1. In 2006 United States of America attacked
Which of the following statement is correct? Lebanon to control the militant group Hezbollah.
a) 1 only 2. The UN passed a resolution on this only in August
b) 2 only and the army withdrew from the region only in
c) Both 1 and 2 October.
d) Neither 1 nor 2 Which of the following statements is/are correct?
a) 1 only
Q.70) Unfortunately recognizing the need for co- b) 2 only
operation and actually co-operating are two different c) Both 1 and 2
things. Keeping in view the above statement which d) Neither 1 nor 2
among the following is/are correct?
1. Nations cannot always agree on how best to Q.74) Consider the following statements regarding
cooperate how to share the costs of co-operating Human rights watch:
and how to ensure that others do not break or 1. Human rights watch is an intergovernmental body
cheat on an agreement. involved in research and advocacy on human
2. An International Organisation can provide rights.
mechanisms rules and a bureaucracy to honour 2. It helped in building international coalitions like
the terms and conditions of an agreement. the campaigns to ban landmines and to stop the
3. International Organisations help member states use of child soldiers.
to resolve their problems peacefully. Which of the following statements is/are correct?
Select the correct option using code below: a) 1 only
a) 1 and 2 only b) 2 only
b) 2 only c) Both 1 and 2
c) 2 and 3 only d) Neither 1 nor 2
d) All of the above
Q.75) Consider the following statements:
Q.71) Consider the following statements with respect 1. Human security is about the protection of people
to Non aligned movement: more than the protection of states and secure
1. Non alignment allowed India to take international states automatically mean secure peoples.
decisions rather than superpowers. 2. Terrorism refers to political violence that targets
2. India’s non alignment was said to be unprincipled. civilians deliberately and indiscriminately.
Which of the following statements is/are correct? Which of the following statements is/are correct?
a) 1 only a) 1 only

@upscmaterialonline
NCERT CLASS XII CONTEMPORARY WORLD POLITICS ENGLISH Page 11
UPSCMATERIAL.ONLINE

Contact us : info@onlyias.com

OnlyIAS Nothing Else Visit : dpp.onlyias.in


Contact : +91-7007 931 912

b) 2 only
c) Both 1 and 2
d) Neither 1 nor 2

@upscmaterialonline
NCERT CLASS XII CONTEMPORARY WORLD POLITICS ENGLISH Page 12
UPSCMATERIAL.ONLINE

Contact us : info@onlyias.com

OnlyIAS Nothing Else Visit : dpp.onlyias.in


Contact : +91-7007 931 912

Q.1) Ans: c leaders of eighteen countries in the East Asian


Exp: region, with ASEAN in a leadership position.
 Bulgaria, Slovak, Eastern Germany, Poland were Membership was initially all ten members of
some of the non-Asian members of Warsaw pact. ASEAN plus China, Japan, South Korea, India,
 Statement 1 is correct: Warsaw Pact was created Australia, and New Zealand, but was expanded to
in reaction to the integration of West Germany include the United States and Russia at the Sixth
into NATO .It was basically the military EAS in 2011.
complement to the Council for Mutual Economic  Statement 2 is correct: In 2006, ASEAN was given
Assistance (CoMEcon), the regional economic observer status at the United Nations General
organization for the communist states of Central Assembly. In response, the organisation awarded
and Eastern Europe. the status of "dialogue partner" to the UN.
 Statement 2 is correct: Indo-Soviet Treaty of  Statement 3 is incorrect: The 'ASEAN Way' refers
Friendship, 1971 was signed in the follow up to to a methodology or approach to solving issues
the Bangladeshi liberation war when India needed that respects the cultural norms of Southeast
military support of Russia. It was argued that this Asia. It is described as a working process or style
effectively diluted the NAM credentials of India, that is informal and personal.
but India never officially accepted the Warsaw
position maintained an official NAM position. Q.4) Ans : d
Exp:
Q.2) Ans: b  Statement 1 is incorrect: The Universal
Exp: Declaration of Human Rights (UDHR) was
 Statement 1 is incorrect: It was envisaged to give adopted by the UNGA in 1948 itself, whereas the
the LDCs and developing countries a more Amnesty International was found in 1961. So, 1
equitable Role in the international economic must clearly be wrong.
system.  Statement 2 is incorrect: In the field of
 Statement 3 is correct: Developing countries must international human rights organisations,
be entitled to regulate and control the activities of Amnesty has the second longest history, after the
multinational corporations operating within their International Federation for Human Rights and
territory. broadest name recognition, and is believed by
 They must be free to nationalize or expropriate many to set standards for the movement as a
foreign property on conditions favourable to whole.
them.
 Statement 2 is correct: They must be free to set Q.5) Ans : a
up associations of primary commodities Exp:
producers similar to the OPEC; all other States  Statement 1 is correct: It was a meeting of Asian
must recognize this right and refrain from taking and African states, most of which were newly
economic, military, or political measures independent. The conference's stated aims were
calculated to restrict it. to promote Afro-Asian economic and cultural
 International trade should be based on the need cooperation and to oppose colonialism or neo-
to ensure stable, equitable, and remunerative colonialism by any nation.
prices for raw materials, generalized non-  Statement 2 is incorrect: The first such summit
reciprocal and non-discriminatory tariff was held in 2008 in New Delhi, India. The Second
preferences, as well as transfer of technology to one in 2011 and the recent one in 2015.
developing countries; and should provide  The Bandung conference was followed by the
economic and technical assistance without any Afro-Asian People's Solidarity Conference in Cairo
strings attached in 1957 and the Belgrade Conference 1961, which
led to the establishment of the Non-Aligned
Q.3) Ans : a Movement.
Exp :
 Statement 1 is correct: The East Asia Summit Q.6) Ans : b
(EAS) is a pan-Asian forum held annually by the Exp:

@upscmaterialonline
NCERT CLASS XII CONTEMPORARY WORLD POLITICS ENGLISH Page 13
UPSCMATERIAL.ONLINE

Contact us : info@onlyias.com

OnlyIAS Nothing Else Visit : dpp.onlyias.in


Contact : +91-7007 931 912

 The cold war saw new defensive alliances, NATO  The idea of a New International Economic Order
in the west and the Warsaw Pact of the Soviet (NIEO) originated with this realisation. The United
satellites. The world was divided into two camps, Nations Conference on Trade and Development
and the "third world" nations were courted and (UNCTAD) brought out a report in 1972 entitled
pressured to join one or the other. The world was Towards a New Trade Policy for Development.
thus divided into two World (poles) which came  The report proposed a reform of the global
to be known as a bipolar world. trading system so as to:
 Statement 1 is correct: USSR and USA emerged o Statement 1 is correct: To give the LDCs
as major powers, and thus the balance of power control over their natural resources
in the world was essentially bipolar. Later, when exploited by the developed Western
the cold war ended, the USSR could not remain a countries.
superpower and USA emerged as the sole o Statement 2 is correct: To obtain access
superpower, resulting in a unipolar world. to Western markets so that the LDCs
 Statement 2 is correct: However, in the modern could sell their products. To make trade
world, many new powers emerged, such as EU, more beneficial for the poorer countries.
China, Japan and the world effectively became o Statement 3 is incorrect: it reduce the
multi-polar in nature. cost of technology from the Western
countries.
Q.7) Ans: d o Statement 4 is correct: To provide the
Exp: LDCs with a greater role in international
 While globalisation is not caused by any single economic institutions.
factor, technology remains a critical element.
 Statement 1 is correct: There is no doubt that the Q.10) Ans: d
invention of the telegraph, the telephone, and the Exp:
microchip in more recent times has  Statement 1 is correct: A coup took place in 1991
revolutionised communication between actors in that was encouraged by Communist Party
different parts of the world. hardliners. The people had tasted freedom by
 Statement 2 is correct: When printing initially then and did not want the old-style rule of the
came into being it laid the basis for the creation of Communist Party.
nationalism.  Statement 2 is correct: Boris Yeltsin emerged as a
 Statement 3 is correct: So also today we should national hero in opposing this coup. The Russian
expect that technology will affect the way we Republic, where Yeltsin won a popular election,
think of our personal but also our collective lives. began to shake off centralised control. Power
began to shift from the Soviet centre to the
Q 8) Ans: c republics, especially in the more Europeanised
Exp: part of the Soviet Union, which saw themselves
 Statement 1 is correct: World Social Forum (WSF) as sovereign states.
is another global platform, which brings together  The Central Asian republics did not ask for
a wide coalition composed of human rights independence and wanted to remain with the
activists, environmentalists, labour, youth and Soviet Federation.
women activists opposed to neo-liberal  Statement 3 is correct: It December 1991, under
globalisation. the leadership of Yeltsin, Russia, Ukraine and
 Statement 2 is incorrect: The first WSF meeting Belarus, three major republics of the USSR,
was organised in Porto Alegre, Brazil in 2001. declared that the Soviet Union was disbanded.
 Statement 3 is incorrect: The fourth WSF meeting  The Communist Party of the Soviet Union was
was held in Mumbai in 2004. The seventh WSF banned. Capitalism and democracy were adopted
meeting was held in Nairobi, Kenya in January as the bases for the post-Soviet republics.
2007 LEADERS OF THE SOVIET UNION Joseph Stalin
(1879-1953) Successor to Lenin and led the
Q.9) Ans: b Soviet Union during its consolidation (1924-53);
Exp: began rapid industrialisation and forcible

@upscmaterialonline
NCERT CLASS XII CONTEMPORARY WORLD POLITICS ENGLISH Page 14
UPSCMATERIAL.ONLINE

Contact us : info@onlyias.com

OnlyIAS Nothing Else Visit : dpp.onlyias.in


Contact : +91-7007 931 912

collectivisation of agriculture; credited with  Statement 1 is incorrect: In 2003, ASEAN moved


Soviet victory in the Second World War; held along the path of the EU by agreeing to establish
responsible for the Great Terror of the 1930s, an ASEAN Community comprising three pillars,
authoritarian functioning and elimination of namely, the ASEAN Security Community, the
rivals within the party. ASEAN Economic Community and The ASEAN
Socio-Cultural Community.
Q.11) Ans: d  The ASEAN security community was based on the
Exp: conviction that outstanding territorial disputes
 Statement 1 is incorrect: European integration should not escalate into armed confrontation. By
after 1945 was aided by the Cold War. America 2003, ASEAN had several agreements in place by
extended massive financial help for reviving which member states promised to uphold peace,
Europe’s economy under what was called the neutrality, cooperation, non-interference, and
‘Marshall Plan’. respect for national differences and sovereign
 Statement 2 is incorrect: The US also created a rights.
new collective security structure under NATO.  Statement 2 is correct: ASEAN Regional Forum
Under the Marshall Plan, the Organisation for (ARF), which was established in 1994, is the
European Economic Cooperation (OEEC) was organisation that carries out coordination of
established in 1948 to channel aid to the west security and foreign policy.
European states.
 Statement 3 is incorrect: It became a forum Q.14) Ans: a
where the western European states began to Exp:
cooperate on trade and economic issues. The  Statement 2 is correct: India and Pakistan also
Council of Europe, established in 1949, was have had problems over the sharing of river
another step forward in political cooperation. waters. Until 1960, they were locked in a fierce
argument over the use of the rivers of the Indus
Q.12) Ans: d basin. Eventually, in 1960, with the help of the
Exp: World Bank, India and Pakistan signed the Indus
 The Union of Soviet Socialist Republics (USSR) Waters Treaty which has survived to this day in
came into being after the socialist revolution in spite of various military conflicts in which the two
Russia in 1917. countries have been involved.
 Statement 1 is correct: The revolution was  Statement 1 is incorrect: Nepal and India enjoy a
inspired by the ideals of socialism, as opposed to very special relationship that has very few
capitalism, and the need for an egalitarian society. parallels in the world. A treaty between the two
 Statement 3 is correct: This was perhaps the countries allows the citizens of the two countries
biggest attempt in human history to abolish the to travel to and work in the other country without
institution of private property and consciously visas.
design a society based on principles of equality.
 Statement 2 is correct: The makers of the Soviet Q.15) Ans: b
system gave primacy to the state and the Exp:
institution of the party.  Statement 1 is incorrect: The states of South Asia
 The Soviet political system centred around the recognise the importance of cooperation and
communist party, and no other political party or friendly relationship, among themselves. The
opposition was allowed. The economy was South Asian Association for Regional Cooperation
planned and controlled by the state. (SAARC) is a major regional initiative by the South
Asian states to evolve cooperation through
Q.13) Ans: a multilateral means.
Exp:  Statement 2 is correct: External influence of non-
 With some of the fastest growing economies in regional players is a major bone of contention
the world, SEAN broadened its objectives beyond which prevents attainment of full potential of
the economic and social spheres. SAARC.

@upscmaterialonline
NCERT CLASS XII CONTEMPORARY WORLD POLITICS ENGLISH Page 15
UPSCMATERIAL.ONLINE

Contact us : info@onlyias.com

OnlyIAS Nothing Else Visit : dpp.onlyias.in


Contact : +91-7007 931 912

Q.16) Ans: a and developing countries of the Third World,


Exp: called the ‘global South’.
 Statement 1 is correct: Throughout the world,  Statement 3 is correct: the Northern states were
cultivable area is barely expanding any more, and concerned with ozone depletion and global
a substantial portion of existing agricultural land is warming, the Southern states were anxious to
losing fertility. address the relationship between economic
 Grasslands have been overgrazed and fisheries development and environmental management.
overharvested. Water bodies have suffered
extensive depletion and pollution, severely Q.18) Ans: a
restricting food production. Exp:
 According to the Human Development Report  Statement 1 is correct: Globalisation has involved
2006 of the United Nations Development greater trade in commodities across the globe;
Programme, 1.2 billion people in developing the restrictions imposed by different countries on
countries have no access to safe water and 2.6 allowing the imports of other countries have been
billion have no access to sanitation, resulting in reduced.
the death of more than three million children  Statement 2 is incorrect: Similarly, the
every year. restrictions on movement of capital across
 Statement 2 is correct: the steady decline in the countries have also been reduced. In operational
total amount of ozone in the Earth’s stratosphere terms, it means that investors in the rich countries
(commonly referred to as the ozone hole) poses a can invest their money in countries other than
real danger to ecosystems and human health. their own, including developing countries, where
 Statement 3 is incorrect: Coastal pollution too is they might get better returns. Globalisation has
increasing globally. Although the open sea is also led to the flow of ideas across national
relatively clean, the coastal waters are becoming boundaries.
increasingly polluted largely due to land-based  The spread of internet and computer related
activities. If unchecked, intensive human services is an example of that. But globalisation
settlement of coastal zones across the globe will has not led to the same degree of increase in the
lead to further deterioration in the quality of movement of people across the globe.
marine environment.  Statement 3 is correct: Developed countries have
carefully guarded their borders with visa policies
Q.17) Ans: d to ensure that citizens of other countries cannot
Exp: take away the jobs of their own citizens
 The growing focus on environmental issues within
the arena of global politics was firmly Q.19) Ans: b
consolidated at the United Nations Conference on Exp:
Environment and Development held in Rio de  Statement 1 is correct: International Monetary
Janeiro, Brazil, in June 1992. This was also called Fund (IMF) is an international organisation that
the Earth Summit. oversees those financial institutions and
 Statement 1 is correct: The summit was attended regulations that act at the international level.
by 170 states, thousands of NGOs and many The IMF has 190 members countries, but they
multinational corporations. do not enjoy an equal say.
 Statement 2 is correct: five years earlier, the  Statement 2 is correct: the top ten countries
1987 Brundtland Report, Our Common Future, have 55 per cent of the votes. They are the
had warned that traditional patterns of economic G-8 members (the US, Japan, Germany, France,
growth were not sustainable in the long term, the UK, Italy and Canada) ,Saudi Arabia and
especially in view of the demands of the South for China. The US alone has 17.4 per cent voting
further industrial development. What was obvious rights.
at the Rio Summit was that the rich and  Statement 3 is incorrect: IMF is an independent
developed countries of the First World, generally organization. It is not an specialised agency of UN.
referred to as the ‘global North’ were pursuing a
different environmental agenda than the poor Q.20) Ans: a

@upscmaterialonline
NCERT CLASS XII CONTEMPORARY WORLD POLITICS ENGLISH Page 16
UPSCMATERIAL.ONLINE

Contact us : info@onlyias.com

OnlyIAS Nothing Else Visit : dpp.onlyias.in


Contact : +91-7007 931 912

Exp:
Option a is correct Q.23) Ans: b
 India is the second most populous country in the Exp:
world comprising almost one-fifth of the world  Statement 1 is correct: Hegemony is about the
population. Moreover, India is also the world’s capacity to ‘manufacture consent’. Here,
largest democracy. hegemony implies class ascendancy in the social,
 India has participated in virtually all of the political and particularly ideological spheres.
initiatives of the UN. Its role in the UN’s Hegemony arises when the dominant class or
peacekeeping efforts is a long and substantial country can win the consent of dominated
one. The country’s economic emergence on the classes, by persuading the dominated classes to
world stage is another factor that perhaps justifies view the world in a manner favourable to the
India’s claim to a permanent seat in the Security ascendancy of the dominant class.
Council. India has also made regular financial  Statement 2 is correct: Adapted to the field of
contributions to the UN and never faltered on its world politics, this notion of hegemony suggests
payments. that a dominant power deploys not only military
 India is aware that permanent membership of the power but also ideological resources to shape the
Security Council also has symbolic importance. It behaviour of competing and lesser powers.
signifies a country’s growing importance in world  Statement 3 is incorrect: The behaviour of the
affairs. This greater status is an advantage to a weaker countries is influenced in ways that favour
country in the conduct of its foreign policy: the the interests of the most powerful country, in
reputation for being powerful makes you more particular its desire to remain preeminent.
influential. Consent, in other words, goes hand-in-hand
with, and is often more effective than, coercion.
Q.21) Ans: a
Exp: Q.24) Ans: c
Option a is correct Exp:
The MDGs are drawn from the actions and targets  Statement 1 is incorrect: Britain’s Prime Minister
contained in the Millennium Declaration that was Tony Blair was America’s partner in the Iraq
adopted by 189 nations-and signed by 147 heads of invasion, and many of the EU’s newer members
state and governments during the UN Millennium made up the US-led ‘coalition of the willing
Summit in September 2000. Full list of Goals, Targets ‘whereas Germany and France opposed
and Indicators are provided below. American policy. There is also a deep-seated
Goal 1: Eradicate extreme poverty and hunger ‘Euro-skepticism’ in some parts of Europe about
Goal 2: Achieve universal primary education the EU’s integrationist agenda.
Goal 3: Promote gender equality and empower  Statement 2 is correct: for example, Britain’s
women former prime minister, Margaret Thatcher, kept
Goal 4: Reduce child mortality the UK out of the European Market.
Goal 5: Improve maternal health  Statement 3 is correct: Denmark and Sweden
Goal 6: Combat HIV/AIDS, malaria and other diseases have resisted the Maastricht Treaty and the
Goal 7: Ensure environmental sustainability adoption of the euro, the common European
Goal 8: Develop a Global Partnership for Development currency. This limits the ability of the EU to act in
matters of foreign relations and defence.
Q.22) Ans: b
Exp: Q.25) Ans: c
Option b is correct Exp:
 US power lies in the overwhelming superiority of  Statement 1 is incorrect: ASEAN was established
its military power. American military dominance in1967 by five countries of this region —
today is both absolute and relative. US Constantly Indonesia, Malaysia, the Philippines, Singapore
trying to gain and retain power in form of military and Thailand — by signing the Bangkok
domination, economic power, political clout and
Declaration.
cultural superiority.

@upscmaterialonline
NCERT CLASS XII CONTEMPORARY WORLD POLITICS ENGLISH Page 17
UPSCMATERIAL.ONLINE

Contact us : info@onlyias.com

OnlyIAS Nothing Else Visit : dpp.onlyias.in


Contact : +91-7007 931 912

 Statement 2 is correct: The objectives of ASEAN  The role of an international organisation can be
were primarily to accelerate economic growth important in this context.
and through that social progress.  Statement 1 is incorrect: An international
 Statement 3 is correct: A secondary objective was organisation is not a super-state with authority
to promote regional peace and stability based on over its members.
the rule of law and the principles of the United  Statement 2 is correct: it is created by and
responds to states.
Nations Charter.
 Statement 3 is correct: It comes into being when
states agree to its creation. Once created, it can
Q.26) Ans: b help member states resolve their problems
Exp: peacefully.
 International organisations are helpful in another
 Statement 2 is correct: Security Council was way. Nations can usually see that there are some
established by the UN Charter in 1945. It is one of things they must do together.
the six principal organs of the United Nations. The  There are issues that are so challenging that they
Security Council was established by the UN can only be dealt with when everyone works
Charter in 1945. It is one of the six principal together
organs of the United Nations.
 The other 5 organs of the United Nations are—the Q.28) Ans: c
General Assembly, the Trusteeship Council, the
Exp:
Economic and Social Council, the International
Court of Justice, and the Secretariat.  Statement 1 is incorrect: International
 Its primary responsibility is to work to maintain Monetary Fund (IMF) is an international
international peace and security. organisation that oversees those financial
 Statement 1 is incorrect: council has 15 institutions and regulations that act at the
members: the five permanent members and 10 international level.
non-permanent members elected for two-year  Statement 2 is incorrect: IMF has 184 members
terms. countries, but they do not enjoy an equal say.
 The five permanent members are the United The top ten countries have 55 per cent of the
States, the Russian Federation, France, China and votes. They are the G-8 members (the US,
the United Kingdom. Japan, Germany, France, the UK, Italy, Canada
 Each member of the Security Council has one and Russia), Saudi Arabia and China. The US
vote. Decisions of the Security Council on matters alone has 17.4 percent voting rights.
are made by an affirmative vote of nine members
including the concurring votes of the permanent Q.29) Ans: c
members. A “No” vote from one of the five
permanent members blocks the passage of the Exp:
resolution.  Statement 1 is correct: UN was founded as a
 Any member of the United Nations which is not a successor to the League of Nations.
member of the Security Council may participate,  Statement 2 is incorrect: was established in 1945
without vote, in the discussion of any question immediately after the Second World War.
brought before the Security Council whenever the  The organisation was set up through the signing
latter considers that the interests of that member of the United Nations Charter by 51 states. It tried
are specially affected. to achieve what the League could not between
 The council's presidency is a capacity that rotates the two world wars.
every month among its 15 members.  The UN’s objective is to prevent international
 The council is headquartered at New York. conflict and to facilitate cooperation among
states.
Q.27) Ans: b  Statement 3 is correct: It was founded with the
Exp: hope that it would act to stop the conflicts
between states escalating into war and, if war

@upscmaterialonline
NCERT CLASS XII CONTEMPORARY WORLD POLITICS ENGLISH Page 18
UPSCMATERIAL.ONLINE

Contact us : info@onlyias.com

OnlyIAS Nothing Else Visit : dpp.onlyias.in


Contact : +91-7007 931 912

broke out, to limit the extent of hostilities.  It settles legal disputes between States and gives
Furthermore, since conflicts often arose from the advisory opinions in accordance with international
lack of social and economic development, the UN law, on legal questions referred to it by
was intended to bring countries together to authorized United Nations organs and specialized
improve the prospects of social and economic agencies.
development all over the world.  It has 193 state parties and current President is
Ronny Abraham.
 Statement 3 is incorrect: The Court is composed
Q.30) Ans: c
of 15 judges, who are elected for terms of office
Exp: of nine years by the United Nations General
Assembly and the Security Council. These organs
 Statement 1 is correct: The name “United vote simultaneously but separately.
Nations”, coined by United States President
Franklin D. Roosevelt. A document called The
Declaration by United Nations was signed in 1942 Q.32) Ans: d
by 26 nations, pledging their Governments to
Exp:
continue fighting together against the Axis Powers
(Rome-Berlin-Tokyo Axis) and bound them against  Statement 1 is incorrect: It is assisted by a
making a separate peace. Registry, its administrative organ. Its official
 United Nations Conference on International languages are English and French.
Organization (1945) Conference held in San  Statement 2 is incorrect: Unlike other organs of
Francisco (USA), was attended by representatives international organizations, the Court is not
of 50 countries and signed the United Nations composed of representatives of governments.
Charter. Members of the Court are independent judges
 The UN Charter of 1945 is the foundational treaty whose first task, before taking up their duties, is
of the United Nations, as an inter-governmental to make a solemn declaration in open court that
organization. they will exercise their powers impartially and
conscientiously.
Components -The main organs of the UN are
 In order to guarantee his or her independence, no
1. The General Assembly, Member of the Court can be dismissed unless, in
2. The Security Council, the unanimous opinion of the other Members,
3. The Economic and Social Council, he/she no longer fulfils the required conditions.
4. The Trusteeship Council, This has in fact never happened.
5. The International Court of Justice,
6. And the UN Secretariat.
Q.33) Ans: b
 Statement 2 is correct: All the 6 were
established in 1945 when the UN was founded. Exp:

Q.31) Ans: b Trusteeship Council

Exp:  It was established in 1945 by the UN Charter,


under Chapter XIII.
 Statement 1 is correct: was established in 1945
by the United Nations charter and started working  Statement 1 is incorrect: Trust territory is a
in April 1946. non-self-governing territory placed under an
 Statement 2 is incorrect: is the principal judicial administrative authority by the Trusteeship
organ of the United Nations, situated at the Peace Council of the United Nations.
Palace in The Hague (Netherlands).  A League of Nations mandate was a legal status
 Unlike the six principal organs of the United for certain territories transferred from the
Nations, it is the only one not located in New York control of one country to another following
(USA). World War I, or the legal instruments that
contained the internationally agreed-upon

@upscmaterialonline
NCERT CLASS XII CONTEMPORARY WORLD POLITICS ENGLISH Page 19
UPSCMATERIAL.ONLINE

Contact us : info@onlyias.com

OnlyIAS Nothing Else Visit : dpp.onlyias.in


Contact : +91-7007 931 912

terms for administering the territory on behalf  Unlike the Security Council, there is no veto
of the League of Nations. power bestowed to anyone.
 Statement 2 is correct: United Nations trust
territories were the successors of the remaining
League of Nations mandates, and came into Q.35) Ans: c
being when the League of Nations ceased to Exp:
exist in 1946.
 Statement 1 is correct: It is the principal body
 It had to provide international supervision for 11
for coordination, policy review, policy dialogue
Trust Territories that had been placed under the
and recommendations on economic, social and
administration of seven Member States, and
environmental issues, as well
ensure that adequate steps were taken to
as implementation of internationally agreed
prepare the Territories for self-government and
development goals.
independence.
 It has 54 members, elected by the General
 By 1994, all Trust Territories had attained self-
Assembly for overlapping three-year terms.
government or independence. The Trusteeship
Council suspended operation on 1 November  It is the UN's central platform for reflection,
1994. debate and innovative thinking on sustainable
development.
 Each year, ECOSOC structures its work around
Q.34) Ans: c
an annual theme of global importance to
Exp: sustainable development. This ensures focused
attention, among ECOSOC’s array of partners
The UNGA is a principal component of the United and throughout the UN development system.
Nations. It is the main governing body of the
organisation and is also the most representative body  It coordinates the work of the 14 UN specialized
in the UN. agencies, ten functional commissions and five
regional commissions, receives reports from
 Statement 1 is correct. All the members of the UN nine UN funds and programmes and issues
are represented in the UNGA, which is sometimes policy recommendations to the UN system and
referred to as simply the GA. Hence, it has 193 to member states.
members (all the UN member countries).
 Statement 2 is correct :Few important bodies
 Statement 2 is incorrect. It meets annually in its under the purview of ECOSOC:
headquarters in New York City, generally in the
month of September. It can also meet at other
times according to the need. o International labour
Organization (ILO)
 The UNGA is headed by its President, who is
elected for a term of one year. o Food and Agriculture
 Statement 3 is correct. It is sometimes called the Organization (FAO)
parliament of the world. o United Nations Educational, Scientific
 The UNGA deliberates and decides on important and Cultural Organization (UNESCO)
matters such as peace and security, and other
international issues.
 It also decides on the admission of new members. Q.36) Ans: a
 Decisions are taken by voting. Generally, a simple Exp:
majority is considered but in case of important
decisions, a two-thirds majority is considered.  In 1992, the UN General Assembly adopted a
Each member has one vote. resolution. The resolution reflected three main
complaints:

@upscmaterialonline
NCERT CLASS XII CONTEMPORARY WORLD POLITICS ENGLISH Page 20
UPSCMATERIAL.ONLINE

Contact us : info@onlyias.com

OnlyIAS Nothing Else Visit : dpp.onlyias.in


Contact : +91-7007 931 912

 Statement 1 is correct: The Security Council no  The address was known as ‘Atoms for Peace’ and
longer represents contemporary political realities. this was the organisation’s first name when it was
 Statement 2 is incorrect: Its decisions reflect only formally established in 1957.
Western values and interests and are dominated  Headquartered in Vienna, Austria, the IAEA is a
by a few powers. UN agency.
 It lacks equitable representation
 Statement 1 is correct: The primary mandate of
the organisation was and continues to be
Q.37) Ans: c promoting safe, secure and peaceful nuclear
technologies.
Exp:
 Currently, it has 171 members. The latest member
 The following steps should be taken to make the is Saint Lucia which joined the IAEA in 2019.
UN more relevant in the changing context:  India became a member in 1957 itself.
o Creation of a Peacebuilding Commission.
 By ensuring the peaceful usage of nuclear
o Statement 2 is correct: Acceptance of the
technologies, the IAEA contributes to peace and
responsibility of the international
security in the world and also towards achieving
community in case of failures of national
the Sustainable Development Goals (SDGs).
governments to protect their own citizens
from atrocities .  The current Director-General of the organisation
o Establishment of a Human Rights Council. is Rafael Mariano Grossi.
o Statement 1 is correct: Agreements to  Statement 2 is correct: IAEA teams regularly
achieve the Millennium Development inspect nuclear facilities all over the world to
Goals. ensure that civilian re ac tors are not being
o Condemnation of terrorism in all its used for military purposes.
forms and manifestations Creation of a  Although the IAEA is an independent international
Democracy Fund. organisation, it reports annually to the UNGA.
o An agreement to wind up the Trusteeship
 The IAEA has regional offices in Geneva, New
Council
York, Toronto and Tokyo; and research
Q.38) Ans: b laboratories in Austria, Italy and Monaco.
Exp:
 Statement 1 is incorrect : Amnesty International Q.40) Ans: a
is an NGO that campaigns for the protection of Exp:
human rights allover the world.
 Statement 2 is correct: It promotes respect for all  The World Trade Organization (WTO) is an
the human rights in the Universal Declaration of international organization which sets the rules
Human Rights. It believes that human rights are for global trade.
interdependent and indivisible.  Statement 1 is correct: This organization was set
 Statement 3 is correct: It prepares and publishes up in 1995 as the successor to the General
reports on human rights. Agreement on Trade and Tariffs (GATT) created
 Governments are not always happy with these after the Second World War.
reports since a major focus of Amnesty is the  Statement 2 is incorrect: It has 164 members. All
misconduct of government authorities. decisions are taken unanimously but the major
 Nevertheless, these reports play an important economic powers such as the US, EU and
role in research and advocacy on human rights. Japan have managed to use the WTO to
frame rules of trade to advance their own
interests.
Q.39) Ans: c  The developing countries often complain of
Exp: non-transparent procedures and being pushed
around by big powers.

@upscmaterialonline
NCERT CLASS XII CONTEMPORARY WORLD POLITICS ENGLISH Page 21
UPSCMATERIAL.ONLINE

Contact us : info@onlyias.com

OnlyIAS Nothing Else Visit : dpp.onlyias.in


Contact : +91-7007 931 912

Q.41) Ans: a  The Whips can be rightly said to be the managers


Exp: of the parties within the legislatures.
 Statement 1 is correct. All over the world there is
a tendency in political parties towards the Q.43) Ans: b
concentration of power in one or few leaders at Exp :
the top. Parties do not keep membership  Statement 1 is incorrect: Governor is the nominal
registers, do not hold organisational meetings, head of a state, unlike the Chief Minister who is
and do not conduct internal elections regularly. the real head of a state in India. According to an
 Ordinary members of the party do not get amendment in the Constitution of India (7th
sufficient information on what happens inside the Constitutional Amendment Act), brought about
party. in 1956, the same person can be the Governor of
 They do not have the means or the connections two or more states.
needed to influence the decisions. As a result the Appointment and removal:
Leaders assume greater power to make decisions  The governors and lieutenant-governors are
in the name of the party. appointed by the president for a term of 5 years.
 Since one or few leaders exercise paramount  The term of governor’s office is normally 5 years
power in the party, those who disagree with the but it can be terminated earlier by: Dismissal by
Leadership find it difficult to continue in the party. the president on the advice of the prime minister
More than loyalty to party principles and policies, of the country, at whose pleasure the governor
personal loyalty to the leader becomes more holds office or Resignation by the governor. Thus,
important. the term is subject to pleasure of the president.
 Statement 2 and statement 3 are incorrect. There  Statement 2 is correct: There is no provision of
is constitutional document for the party. There impeachment, as it happens for the president.
are provincial or local branches of the party.  Article 157 and Article 158of the Constitution of
India specify eligibility requirements for the post
Q.42) Ans : c of Governor.
Exp: Powers:
 Statement 1 is correct: The term is derived from  Like the President of India, the Governor of any
the old British practice of “whipping in” state in India is vested with certain executive,
lawmakers to follow the party Line. Parties Legislative and judicial powers.
appoint a senior member from among their House  He or she also possesses certain discretionary or
contingents to issue whips — this member is emergency powers.
called a Chief Whip, and he/she is assisted by  Statement 3 is incorrect: But one major
additional Whips. difference in the powers enjoyed by the President
 Statement 2 is incorrect: Any registered party and those enjoyed by the Governor is, the
can issue a whip. In India, rebelling against a Governor does not have any diplomatic or
three-line whip can put a lawmaker’s membership military powers.
of the House at risk.
 Statement 3 is correct: The anti-defection law Q.44) Ans: c
allows the Speaker/Chairperson to disqualify such Exp:
a member; the only Exception is when more than  Statement 1 is correct: Each tier has its own
a third of legislators vote against a directive, jurisdiction in specific matters of legislation,
effectively splitting the Party. taxation and administration; for e.g. in India
 Importance of whips in our political system: states legislate in matters of police, and Centre
 In the parliamentary form of Government, Whips legislates in areas of national security as a whole.
of various political parties are the vital links of the  Tiers may be dependent upon each other;
Internal organization of parties, inside the however not completely, else it erodes autonomy.
legislatures. The efficient and smooth functioning  Statement 2 is correct: The fundamental
of Parliament and State Legislatures depends, to a provisions of the constitution cannot be
considerable extent, upon the office of the Whip. unilaterally changed by one level of government.

@upscmaterialonline
NCERT CLASS XII CONTEMPORARY WORLD POLITICS ENGLISH Page 22
UPSCMATERIAL.ONLINE

Contact us : info@onlyias.com

OnlyIAS Nothing Else Visit : dpp.onlyias.in


Contact : +91-7007 931 912

Such changes require the consent of both the vulnerable communities of developing countries,
levels of government. serving 5.8 million direct beneficiaries.

Q.45) Ans: d Q.48) Ans: a


Exp: Exp:
 Constitution is the supreme law that determines  Statement 1 is correct: IPCC was created
the relationship among citizens and also the to provide policymakers with regular scientific
relationship between the citizens and assessments on climate change
government.  Statement 2 is correct: its implications and
 Statement 1 is correct: First, it generates a potential future risks, as well as to put forward
degree of trust and coordination that is necessary adaptation and mitigation options. The IPCC does
for different kind of people to live together. not conduct its own research.
 Statement 2 is correct: Second, it specifies how  Statement 3 is incorrect: The assessment reports
the government will be constituted, for e.g. by are a key input into the international negotiations
democratic elections. to tackle climate change. Created by the United
 Third, it lays down limits on the powers of the Nations Environment Programme (UN
government and tells us what the rights of the Environment) and the World Meteorological
citizens together. Organization (WMO) in 1988, the IPCC has 195
 Statement 3 is correct: Fourth, it expresses the members.
aspirations of the people, for e.g. by preamble
and DPSP. Q.49) Ans: b
Exp:
Q.46) Ans : c Option b is correct:
Exp:  The World Bank serves as the GEF Trustee,
 Statement 1 is correct: This is called public administering the GEF Trust Fund (contributions
interest litigation. The courts intervene to prevent by donors). The Trustee helps mobilize GEF
the misuse of the government’s power to make resources; disburses funds to GEF Agencies;
decisions. For e.g. the first PIL was filed on the prepares financial reports on investments and use
poor living condition of the jail inmates, the court of resources; and monitors application of
took immediate action on the issue. budgetary and project funds.
 Statement 2 is correct: Courts can issue writs of  Bhutan’s foreign ministry has issued a demarche
Mandamus to warn public officials of lapse of to the Chinese embassy in New Delhi for the
duty and unruly behaviour. They can also order claims made by Beijing over Sakteng Wildlife
them to perform certain functions, for e.g. sanctuary, situated in eastern Bhutan.
ensuring passenger safety on roads or safety of  Bhutan’s western and middle sector have been in
women at workplaces (Vishakha guidelines). dispute with China (Jakarlung, Pasamlung and
Chumbi Valley). However, the eastern sector has
Q.47) Ans: c not been part of the boundary talks and China had
Exp: not claimed rights over Sakteng wildlife sanctuary
Option C is correct earlier.
 It is established under the Kyoto Protocol of the  The recent claim was made at the 58th meeting of
UN Framework Convention on Climate Change. the Global Environment Facility (GEF) Council
 It finances projects and programmes that help where China tried to “oppose” funding to a
vulnerable communities in developing countries project for the Sakteng Wildlife Sanctuary situated
adapt to climate change. in Bhutan saying that it was “disputed” territory.
 Initiatives are based on country needs, views and
priorities. Q.50) Ans: d
 The latest data show that since 2010, the Exp:
Adaptation Fund has directed $532 million to 80 Option d is correct:
concrete adaptation projects in the most  The ADMM-Plus is a platform for ASEAN and its
eight Dialogue Partners to strengthen security and

@upscmaterialonline
NCERT CLASS XII CONTEMPORARY WORLD POLITICS ENGLISH Page 23
UPSCMATERIAL.ONLINE

Contact us : info@onlyias.com

OnlyIAS Nothing Else Visit : dpp.onlyias.in


Contact : +91-7007 931 912

defence cooperation for peace, stability, and  Statement 2 is correct: As per Article 9 of the
development in the region. Japanese Constitution, “the Japanese people
 Eight Dialogue Partners are Australia, China, India, forever renounce war as sovereign right of the
Japan, New Zealand, Republic of Korea, Russia and nation and the threat or use of force as means of
the USA (collectively referred to as the “Plus settling international disputes.” It is the only
Countries”). Asian member of the G-7.
 Statement 4 is correct: Although Japan’s military
expenditure is only one per cent of its GDP, it is
Q.51) Ans: c the fourth largest in the world.
Exp:
 Statement 1 is correct: After the inception of the Q.53) Ans: b
People’s Republic of China in 1949, following the Exp:
communist revolution under the leadership of  Statement 1 is incorrect: In August 1990, Iraq
Mao, its economy was based on the Soviet invaded Kuwait, rapidly occupying and
model. subsequently annexing it. After a series of
 Statement 2 is incorrect: The economically diplomatic attempts failed at convincing Iraq to
backward communist China chose to sever its quit its aggression, the United Nations mandated
links with the capitalist world. It had little choice the liberation of Kuwait by force.
but to fall back on its own resources and, for a  Statement 2 is correct: For the UN, this was a
brief period, on Soviet aid and advice. The model dramatic decision after years of deadlock during
was to create a state-owned heavy industries the Cold War. The US President George H.W.
sector from the capital accumulated from Bush hailed the emergence of a ‘new world
agriculture. As it was short of foreign exchange order’.
that it needed in order to buy technology and  Statement 3 is correct: A massive coalition force
goods on the world market, China decided to of 660,000 troops from 34 countries fought
substitute imports by domestic goods. against Iraq and defeated it in what came to be
 Statement 3 is correct: This model allowed China known as the First Gulf War.
to use its resources to establish the foundations
of an industrial economy on a scale that did not Q.54) Ans: d
exist before. Employment and social welfare was Exp:
assured to all citizens, and China moved ahead of  Statement 1 is correct : The idea of a New
most developing countries in educating its citizens International Economic Order (NIEO) originated
and ensuring better health for them. The with this realisation. The United Nations
economy also grew at a respectable rate of 5-6 Conference on Trade and Development (UNCTAD)
per cent. But an annual growth of 2-3 per cent in brought out a report in 1972 entitled Towards a
population meant that economic growth was New Trade Policy for Development.
insufficient to meet the needs of a growing  Statement 2 is correct: It was meant to be a
population. Agricultural production was not revision of the international economic system in
sufficient to generate a surplus for industry. favour of Third World countries replacing the
Bretton Woods system.
Q.52) Ans: d  Statement 3 is correct: The late 1980s, the NIEO
Exp: initiative had faded, mainly because of the stiff
 Statement 1 is correct: Japan is the only nation opposition from the developed countries who
that suffered the destruction caused by nuclear acted as a united group while the non-aligned
bombs. Japan has very few natural resources and countries struggled to maintain their unity in the
imports most of its raw materials. face of this opposition.
 Statement 3 is correct: It is the second largest
contributor to the regular budget of the UN, Q.55) Ans: a
contributing almost 20 per cent of the total. Japan Exp:
has a security alliance with the US since 1951.  Statement 1 is correct: LIMITED TEST BAN TREATY
(LTBT)Banned nuclear weapon tests in the

@upscmaterialonline
NCERT CLASS XII CONTEMPORARY WORLD POLITICS ENGLISH Page 24
UPSCMATERIAL.ONLINE

Contact us : info@onlyias.com

OnlyIAS Nothing Else Visit : dpp.onlyias.in


Contact : +91-7007 931 912

atmosphere, in outer space and under water.  Statement 2 is correct: The party refused to
Signed by the US, UK and USSR in Moscow on 5 recognise the urge of people in the fifteen
August 1963.Entered into force on 10 October different republics that formed the Soviet Union
1963. to manage their own affairs including their
 Statement 2 is incorrect: NUCLEAR NON- cultural affairs.
PROLIFERATION TREATY (NPT)Allows only the  Statement 3 is correct: Although, on paper, Russia
nuclear weapon states to have nuclear weapons was only one of the fifteen republics that together
and stops others from acquiring them. For the constituted the USSR, in reality Russia dominated
purposes of the NPT, a nuclear weapon state is everything, and people from other regions felt
one which has manufactured and exploded a neglected and often suppressed. In the arms race,
nuclear weapon or other nuclear explosive device the Soviet Union managed to match the US from
prior to 1 January 1967. So there are five nuclear time to time, but at great cost. The Soviet Union
weapon states: lagged behind the West in technology,
 US, USSR (later Russia), Britain, France and China. infrastructure (e.g. transport, power), and most
Signed in Washington, London, and Moscow on 1 importantly, in fulfilling the political or economic
July 1968. aspirations of citizens.
 Entered into force on 5 March 1970. Extended  Statement 4 is correct: The Soviet invasion of
indefinitely in 1995. Afghanistan in 1979 weakened the system even
further. Though wages continued to grow,
Q.56) Ans: d productivity and technology fell considerably
Exp: behind that of the West. This led to shortages in
 Statement 1 is incorrect: STRATEGIC ARMS all consumer goods. Food imports increased every
REDUCTION TREATY II (START-II)Treaty signed by year. The Soviet economy was faltering in the late
the Russian President Boris Yeltsin and the US 1970s and became stagnant.
President George Bush (Senior)on the reduction
and limitation of strategic offensive arms in Q.59) Ans: c
Moscow on 3 January 1993. Exp:
 Statement 2 is incorrect: STRATEGIC ARMS  Statement 3 is incorrect: Russia, Belarus and
LIMITATION TALKS II (SALT-II)The second round Ukraine decide to annul the 1922 Treaty on the
started in November 1972. The US President Creation of the USSR and establish the
Jimmy Carter and the Soviet leader Leonid Commonwealth of Independent States (CIS);
Brezhnev signed the Treaty on the limitation of Armenia, Azerbaijan, Moldova, Kazakhstan,
strategic offensive arms in Vienna on 18 June Kyrgyzstan, Tajikistan, Turkmenistan and
1979. Uzbekistan join the CIS (Georgia joins later in
1993); Russia takes over the USSR seat in the
Q.57) Ans: a United Nations.
Exp:  Statement 1 is incorrect: Lithuania becomes the
Option a is correct: first of the 15 Soviet republics to declare its
1.Korean war resulting in division of Korea- independence 1990 February.
2. Signing of the Baghdad pact  Statement 2 is correct: Gorbachev strips the
3. American intervention in Vietnam Soviet Communist Party of its 72-year-
4. Cuban missile crisis longmonopoly on power by calling on the Soviet
5. Formation of SEATO parliament (Duma) to permit multi-party politics
in 1990.
Q.58) Ans: d
Exp: Q.60) Ans: c
 Statement 1 is correct: The one-party system Exp:
represented by the Communist Party of the Soviet  Statement 1 is correct: Leader of the Soviet
Union had tight control over all institutions and Union (1964-82); proposed Asian Collective
was unaccountable to the people. Security system; associated with the détente
phase in relations with the US; involved in

@upscmaterialonline
NCERT CLASS XII CONTEMPORARY WORLD POLITICS ENGLISH Page 25
UPSCMATERIAL.ONLINE

Contact us : info@onlyias.com

OnlyIAS Nothing Else Visit : dpp.onlyias.in


Contact : +91-7007 931 912

suppressing a popular rebellion in controlled the development of the region through


Czechoslovakia and in invading Afghanistan. various agencies and organisation.
European integration after 1945 was aided by the
Cold War. The Truman Doctrine of containment Q.63) Ans: a
was a United States policy to stop Soviet Exp:
expansion during the Cold War. America extended  Statement 1 is correct: Russia, started reviving in
massive financial help for reviving Europe’s 2000, ten years after their independence. The
economy under what was called the ‘Marshall reason for the revival for most of their economies
Plan’. was the export of natural resources like oil,
 Statement 2 is correct: The US also created a new natural gas and minerals. Azerbaijan, Kazakhstan,
collective security structure under NATO.The Russia, Turkmenistan and Uzbekistan are major oil
Marshall Plan was the American initiative to aid and gas producers.
Europe.  Statement 2 is correct: Other countries have
gained because of the oil pipelines that cross their
Q.61) Ans: d territories for which they get rent. Some amount
Exp: of manufacturing has restarted.
 Statement 2 is correct: In eastern Europe,  Statement 3 is incorrect: The construction of
Czechoslovakia split peacefully into two, with the democratic institutions was not given the same
Czechs and the Slovaks forming independent attention and priority as the demands of
countries. economic transformation. The constitutions of all
 But the most severe conflict took place in the these countries were drafted in a hurry and most,
Balkan republics of Yugoslavia. including Russia, had a strong executive president
 Statement 1 is correct: After 1991, it broke apart with the widest possible powers that rendered
with several provinces like Croatia, Slovenia and elected parliaments relatively weak. In Central
Bosnia and Herzegovina declaring independence. Asia, the presidents had great powers, and several
 Ethnic Serbs opposed this, and a massacre of non- of them became very authoritarian. For example,
Serb Bosnians followed. the presidents of Turkmenistan and Uzbekistan
 The NATO intervention and the bombing of appointed themselves to power first for ten years
Yugoslavia followed the inter-ethnic civil war. and then extended it for another ten years.
 Statement 4 is incorrect: They allowed no dissent
Q.62) Ans: c or opposition. A judicial culture and
Exp: independence of the judiciary was yet to be
 Shock therapy also involved a drastic change in established in most of these countries.
the external orientation of these economies.
Development was now envisaged through more Q.64) Ans: c
trade, and thus a sudden and complete switch to Exp:
free trade was considered essential.  Statement 1 is incorrect: India has maintained
 Statement 1 is correct: The free trade regime and good relations with all the post communist
foreign direct investment (FDI) were to be the countries. But the strongest relations are still
main engines of change. This also involved those between Russia and India. India’s relations
openness to foreign investment, financial opening with Russia are an important aspect of India’s
up or deregulation, and currency convertibility. foreign policy.
 Statement 2 is incorrect: Finally, the transition  Statement 2 is correct: The Soviet Union
also involved a break up of the existing trade assisted India’s public sector companies at a
alliances among the countries of the Soviet bloc. time when such assistance was difficult to get.
Each state from this bloc was now linked directly It gave aid and technical assistance for steel
to the West and not to each other in the region. plants like Bhilai, Bokaro, Visakhapatnam, and
 Statement 3 is correct: These states were thus to machinery plants like Bharat Heavy Electricals
be gradually absorbed into the Western economic Ltd., etc .
system. The Western capitalist states now  Statement 3 is correct: India has remained a
became the leaders and thus guided and major arms and oil importer from Russia. The

@upscmaterialonline
NCERT CLASS XII CONTEMPORARY WORLD POLITICS ENGLISH Page 26
UPSCMATERIAL.ONLINE

Contact us : info@onlyias.com

OnlyIAS Nothing Else Visit : dpp.onlyias.in


Contact : +91-7007 931 912

Soviet Union accepted Indian currency for trade  Statement 2 is correct: The Organisation for
when India was short of foreign exchange the Prohibition of Chemical Weapons is the
 India received most of its military hardware implementing body of the Chemical Weapons
from the Soviet Union at a time when few Convention (CWC), which entered into force in
other countries were willing to part with 1997.
military technologies. The Soviet Union entered
into various agreements allowing India to  Statement 1 is incorrect: The OPCW, with its
jointly produce military equipment. 193 member states, has its seat in The
Hague, Netherlands.
Q.65) Ans: d
Exp: Q.67) Ans: a
 The Security Council was established by the UN Exp:
Charter in 1945. It is one of the six principal  International Bank for Reconstruction and
organs of the United Nations. Development (IBRD) and International
 The other 5 organs of the United Nations are—the Development Association (IDA) form the World
General Assembly, the Trusteeship Council, the Bank, which provides financing, policy advice, and
Economic and Social Council, the International technical assistance to governments of developing
Court of Justice, and the Secretariat. countries. While the World Bank Group consists of
 Its primary responsibility is to work to maintain five development institutions.
international peace and security.  International Bank for Reconstruction and
 The council has 15 members: the five permanent Development (IBRD) provides loans, credits, and
members and 10 non-permanent members grants.
elected for two-year terms.  Statement 1 is incorrect: International
 Statement 1 is correct: The five permanent Development Association (IDA) provides low- or
members are the United States, the Russian no-interest loans to low-income countries.
Federation, France, China and the United  Statement 2 is incorrect: The International
Kingdom. Finance Corporation (IFC) provides investment,
 Statement 2 is correct: Each member of the advice, and asset management to companies and
Security Council has one vote. Decisions of the governments.
Security Council on matters are made by an  Statement 3 is correct: Multilateral Guarantee
affirmative vote of nine members including the Agency (MIGA) insures lenders and investors
concurring votes of the permanent members. A against political risk such as war.
“No” vote from one of the five permanent  The International Centre for the Settlement of
members blocks the passage of the resolution. Investment Disputes (ICSID) settles investment-
 Any member of the United Nations which is not a disputes between investors and countries.
member of the Security Council may participate,  All of these efforts support the Bank Group’s twin
without vote, in the discussion of any question goals of ending extreme poverty by 2030 and
brought before the Security Council whenever the boosting shared prosperity of the poorest 40% of
latter considers that the interests of that member the population in all countries.
are specially affected.
 The council’s presidency is a capacity that rotates Q.68) Ans: b
every month among its 15 members. Exp:
 The council is headquartered at NewYork.  Statement 1 is correct: The World Trade
Organisation (WTO) is an international
Q.66) Ans: a organisation which has sets the rules for global
Exp: trade. This organisation was set up in 1995 as
 Statement 3 is correct: The OPCW is an the successor to the General Agreement on
independent, autonomous international Trade and Tariffs (GATT) created after the
organisation with a working relationship with Second World War.
the United Nations.  Statement 2 is incorrect: It has 150 members. All
decisions are taken unanimously but the major

@upscmaterialonline
NCERT CLASS XII CONTEMPORARY WORLD POLITICS ENGLISH Page 27
UPSCMATERIAL.ONLINE

Contact us : info@onlyias.com

OnlyIAS Nothing Else Visit : dpp.onlyias.in


Contact : +91-7007 931 912

economic powers such as the US, EU and peacefully. International organizations are helpful
Japan have managed to use the WTO to frame in another way.
rules of trade to advance their own interests.  Statement 1 is correct: Nations can usually see
The developing WTO countries often complain that there are some things they must do together.
of non-transparent procedures and being There are issues that are so challenging that they
pushed around by big powers. can only be dealt with when everyone works
 Statement 3 is correct: The decision making together.
power is WTO mostly lies with the top 10
countries having 55% of the votes. Q.71) Ans: c
Exp:
Q.69) Ans: c  Statement 1 is correct: Non-alignment allowed
Exp: India to take international decisions and stances
 SAARC is slowly emerging from the slumbers after that served its interests rather than the interests
caught in between India and Pakistan since of the superpowers and their allies.
inception with new initiatives like SAARC  India was often able to balance one superpower
university. against the other. If India felt ignored or unduly
 Statement 1 is correct: The South Asian pressurised by one superpower, it could tilt
Association for Regional Cooperation (SAARC) is a towards the other. Neither alliance system could
major regional initiative by the South Asian states take India for granted or bully .
to evolve cooperation through multilateral means.  Statement 2 is correct: India’s non-alignment was
It began in 1985. Unfortunately, due to persisting said to be ‘unprincipled’. In the name of pursuing
political differences, SAARC has not had much its national interest, India, it was said, often
success. refused to take a firm stand on crucial
 Statement 2 is correct: SAARC members signed international issues.
the South Asian Free Trade (SAFTA) agreement
which promised the formation of a free trade Q.72) Ans: a
zone for the whole of South Asia. Exp:
 Statement 1 is correct: India and China were
Q.70) Ans: d great powers in Asia before the advent of
Exp: Western imperialism. China had considerable
 Statement 2 is correct: International influence and control on the periphery of its
organizations are not the answer to everything, borders based on its unique tributary system. At
but they are important. International organisation different times in China’s long history of dynastic
help with matters of war and peace. They also rule, Mongolia, Korea, parts of Indo-China, and
help countries cooperate to make better living Tibet accepted China’s authority. Various
conditions for us all. Countries have conflicts and kingdoms and empires in India also extended their
differences with each other. That does not influence beyond their borders.
necessarily mean they must go to war to deal with  Statement 2 is incorrect The conflict of 1962, in
their antagonisms. They can, instead, discuss which India suffered military reverses, had long-
contentious issues and find peaceful solutions; term implications for India–China relations.
indeed, even though this is rarely noticed, most Diplomatic relations between the two countries
conflicts and differences are resolved without were downgraded until 1976. Thereafter,
going to war. relations between the two countries began to
 The role of an international organisation can be improve slowly. After the change in China’s
important in this context. An international political leadership from the mid to late 1970s,
organisation is not a super-state with authority China’s policy became more pragmatic and less
over its members. It is created by and responds to ideological. So it was prepared to put off the
states. settlement of contentious issues while improving
 Statement 3 is correct: It comes into being when relations with India. A series of talks to resolve the
states agree to its creation. Once created, it can border issue were also initiated in 1981.
help member states resolve their problems

@upscmaterialonline
NCERT CLASS XII CONTEMPORARY WORLD POLITICS ENGLISH Page 28
UPSCMATERIAL.ONLINE

Contact us : info@onlyias.com

OnlyIAS Nothing Else Visit : dpp.onlyias.in


Contact : +91-7007 931 912

Q.73) Ans: b
Exp:
 Statement 1 is incorrect: During June 2006, Israel
attacked Lebanon, saying that it was necessary to
control the militant group called Hezbollah. Large
numbers of civilians were killed and many public
buildings and even residential areas came under
Israeli bombardment.
 Statement 2 is correct: The UN passed a
resolution on this only in August and the Israel
army withdrew from the region only in October.

Q.74) Ans: b
Exp:
 Statement 1 is incorrect: Human Rights Watch is
another international NGO involved in research
and advocacy in human rights. It is the largest
international human rights organisation in the US.
It draws the global media’s attention to human
rights abuses.
 Statement 2 is correct: It helped in building
international coalitions like the campaigns to ban
landmines, to stop the use of child soldiers and to
establish the International Criminal Court.

Q.75) Ans: b
Exp:
 Statement 1 is incorrect: Human security is about
the protection of people more than the
protection of states. Human security and state
security should be — and often are — the same
thing. But secure states do not automatically
mean secure peoples.
 Statement 2 is correct: Terrorism refers to
political violence that targets civilians deliberately
and indiscriminately. International terrorism
involves the citizens or territory of more than one
country. Terrorist groups seek to change a
political context or condition that they do not like
by force or threat of force.

@upscmaterialonline
NCERT CLASS XII CONTEMPORARY WORLD POLITICS ENGLISH Page 29

You might also like